1answer.
Ask question
Login Signup
Ask question
All categories
  • English
  • Mathematics
  • Social Studies
  • Business
  • History
  • Health
  • Geography
  • Biology
  • Physics
  • Chemistry
  • Computers and Technology
  • Arts
  • World Languages
  • Spanish
  • French
  • German
  • Advanced Placement (AP)
  • SAT
  • Medicine
  • Law
  • Engineering
Zinaida [17]
3 years ago
14

Which equation does the graph below represent? y=1/3+x y=1/3x y=3+x y=3x

Mathematics
1 answer:
barxatty [35]3 years ago
3 0
Lets find 2 points, one obvious one is (0,0), another point is (12,4)

Slope = \frac{4-0}{12-0} =\frac{4}{12} = \frac{4\times 1}{4\times 3} = \frac{1}{3}

y = mx + b

m is the slope

we got m = 1/3

We can eliminate answer choices C and D.

y = 1/3x + b

we need to solve for b

Plug in the point (0,0) and solve for b

0 = 1/3(0) + b

0 = 0 + b

b = 0

The equation of the line is y = 1/3x + 0, or y = 1/3x

Your answer is B.
You might be interested in
PLEASE HELP!!!! Nicholas is buying shirts for his baseball team. He will pay $8.75 for each shirt plus a one-time fee of $25 for
Marizza181 [45]

Answer:

y = 8.75x + 25

Step-by-step explanation:

8 0
3 years ago
Read 2 more answers
Does this table represent a linear function?
IRINA_888 [86]
I think yes because the number are not all the same
5 0
3 years ago
Which equation represents a line that has a slope of 3/4 and passes through the point (2,1)?
ValentinkaMS [17]
The general equation of a line is:
\frac{y- y_{1}}{x- x_{1}} =m
When m is the gradient and x1 and y1 are the coordinates of the points on the line
Therefore:
\frac{y-1}{x-2}=3/4
4y-4=3x-6
4y=3x-2
5 0
3 years ago
A television network is about to telecast a new television show. Before a show is launched, the network airs a pilot episode and
GalinKa [24]

Let's first denote:

F - a favorable response

F' - an unfavorable response

S - successful

We know that:

P(F) = 0.6\\\\P(F')=0.4\\\\P(F\cap S)=0.5\\\\P(F'\cap S)=0.3

So, from the conditional probability, we can calculate:

P(S|F) = \dfrac{P(S\cap F)}{P(F)} = \dfrac{0.5}{0.6}=\dfrac{5}{6}\approx\boxed{0.83}

Answer E.

5 0
3 years ago
Round each number to the nearest tenth.
andreev551 [17]

Answer:For the first one 26.73 would turn into 27 and 89.42 would stay 89


Step-by-step explanation: Any number that is 5 or greater can be used to round any number.


7 0
3 years ago
Other questions:
  • The ellipse with x-intercepts (2, 0) and (-2, 0); y-intercepts (0, 4) and (0, -4).
    9·1 answer
  • 3(5 √3 + 3 √3) simplified
    10·1 answer
  • Hey guys help my please
    7·1 answer
  • How do you show something declines by a percent%
    10·1 answer
  • Somebody please help!!
    11·1 answer
  • As indicated below, write the equation of the line passing through the point (2,2) and perpendicular to the line whose equation
    8·1 answer
  • How do I solve equations with variable on both sides?
    7·1 answer
  • Add using the number line. ​ −1.2+2.9 ​ Select the location on the number line to plot the point. -2.52.5-2-1.5-1-0.500.511.52.
    10·1 answer
  • Which expression is equivalent to this expression? <br><br> thank you :)
    12·1 answer
  • What is -40/60 as a decimal
    9·2 answers
Add answer
Login
Not registered? Fast signup
Signup
Login Signup
Ask question!